2014 dxdy logo

Научный форум dxdy

Математика, Физика, Computer Science, Machine Learning, LaTeX, Механика и Техника, Химия,
Биология и Медицина, Экономика и Финансовая Математика, Гуманитарные науки


Правила форума


В этом разделе нельзя создавать новые темы.

Если Вы хотите задать новый вопрос, то не дописывайте его в существующую тему, а создайте новую в корневом разделе "Помогите решить/разобраться (М)".

Если Вы зададите новый вопрос в существующей теме, то в случае нарушения оформления или других правил форума Ваше сообщение и все ответы на него могут быть удалены без предупреждения.

Не ищите на этом форуме халяву, правила запрещают участникам публиковать готовые решения стандартных учебных задач. Автор вопроса обязан привести свои попытки решения и указать конкретные затруднения.

Обязательно просмотрите тему Правила данного раздела, иначе Ваша тема может быть удалена или перемещена в Карантин, а Вы так и не узнаете, почему.



Начать новую тему Ответить на тему
 
 задача №1232 из Демидовича
Сообщение08.03.2006, 17:33 


07/01/06
26
$(x^{n-1}*e^{1/x})^{(n)}=\frac {(-1)^n} {x^{n+1}}*e^{1/x}

Это задача №1232 из Демидовича

 Профиль  
                  
 
 Re: Помогите доказать
Сообщение08.03.2006, 18:01 
Аватара пользователя


07/03/06
128
matematikFromMati писал(а):
$(x^{n-1}*e^{1/x})^{(n)}=\frac {(-1)^n} {x^{n+1}}*e^{1/x}
Это задача №1232 из Демидовича

Вам знаком метод математической индукции? Предположим, что утверждение
$$A_n= \left\{(x^{n-1}\cdot e^{1/x})^{(n)}=\frac {(-1)^n} {x^{n+1}}\cdot e^{1/x} \right\}$$
для n-той производной верно, тогда достаточно показать, что будет верно и утверждение $A_{n+1}$.

 Профиль  
                  
 
 
Сообщение08.03.2006, 18:33 


07/01/06
26
Я пробовал доказать по индукции, у меня получилось что если для n верно,

то для n+1 нужно доказать

$-\frac 1 x *(x^{n-1}*e^{1/x})^{(n)}= (x^n * e^{1/x})^{(n+1)}

а это доказать я не могу

 Профиль  
                  
 
 Re: Помогите доказать
Сообщение08.03.2006, 18:46 
Заслуженный участник
Аватара пользователя


17/10/05
3709
:evil:
matematikFromMati писал(а):
$(x^{n-1}*e^{1/x})^{(n)}=\frac {(-1)^n} {x^{n+1}}*e^{1/x}

Это задача №1232 из Демидовича


Воспользуйтесь индукцией два раза. Первый, для того чтобы получить выражение для $(x f(x))^{(k)}$, где $f(x)$ -- произвольная хорошая функция, а $k$ -- натуральное.

Второй -- чтобы вычислить $(x^{n}*e^{1/x})^{(n+1)} = $(x \left(x^{n-1}*e^{1/x}\right))^{(n+1)} (индукционный переход).

 Профиль  
                  
 
 Re: Помогите доказать
Сообщение08.03.2006, 19:23 
Заслуженный участник


09/02/06
4382
Москва
незванный гость писал(а):
:evil:
matematikFromMati писал(а):
$(x^{n-1}*e^{1/x})^{(n)}=\frac {(-1)^n} {x^{n+1}}*e^{1/x}

Это задача №1232 из Демидовича


Воспользуйтесь индукцией два раза.

Достаточно применить индукцию один раз, действительно:
$(x^n*e^{1/x})^{(n+1)}=(nx^{n-1}e^{1/x}-x^{n-2}e^{1/x})^{(n)}=n\frac {(-1)^n}{x^{n+1}}e^{1/x}-(\frac {(-1)^{n-1}}{x^n} e^{1/x})^{'}=\frac{(-1)^{n+1}}{x^{n+2}}e^{1/x}$.
Выполнение при n=0 очевидно.

 Профиль  
                  
 
 
Сообщение08.03.2006, 19:52 


07/01/06
26
Большое спасибо

 Профиль  
                  
 
 Re: Помогите доказать
Сообщение08.03.2006, 21:33 
Аватара пользователя


07/03/06
128
Руст писал(а):
незванный гость писал(а):
:evil:
Воспользуйтесь индукцией два раза.

Достаточно применить индукцию один раз, действительно:
$(x^n*e^{1/x})^{(n+1)}=(nx^{n-1}e^{1/x}-x^{n-2}e^{1/x})^{(n)}=n\frac {(-1)^n}{x^{n+1}}e^{1/x}-(\frac {(-1)^{n-1}}{x^n} e^{1/x})^{'}=\frac{(-1)^{n+1}}{x^{n+2}}e^{1/x}$.
Выполнение при n=0 очевидно.

Тонкость применения индукционного метода в данном примере состоит в том, что удаётся лишь показать истинность следствия $(A_{n-1} \wedge A_n ) \Rightarrow A_{n+1}$ вместо обычного перехода $ A_n  \Rightarrow A_{n+1}$. Отсюда выходит, что проверка истинности одного утверждения $A_0$ недостаточна. В таких случаях необходимо проверять истинность $A_0 \wedge A_1$, то есть обоих случаев для самых первых n. Если бы утверждение $A_1$ в данном примере было бы ложно, то и всё приведённое выше доказательство не имело бы никакой силы. :!:

 Профиль  
                  
 
 Re: Помогите доказать
Сообщение09.03.2006, 18:54 


09/03/06
32
Sibiu ,Romania
matematikFromMati писал(а):
$(x^{n-1}*e^{1/x})^{(n)}=\frac {(-1)^n} {x^{n+1}}*e^{1/x} Это задача №1232 из Демидовича

We have $ f(x):= x^{n-1}e^{\frac{1}{x}}=\sum\limits_{k=0}^{\infty}\frac{x^{n-1-k}}{k!}=
\sum\limits_{k=0}^{n-1}\frac{x^{n-1-k}}{k!}+ \sum\limits_{k=n}^{\infty}\frac{x^{n-1-k}}{k!}=
P(x)+\sum\limits_{k=0}^{\infty}\frac{x^{-1-k}}{(k+n)!}
$
where $ P  $ is a polynomial of degree $  n-1 $ . But $ \left(x^{\alpha}\right)^{(n)}=\alpha(\alpha-1)\cdots(\alpha-n+1)x^{\alpha-n} =
n!{\alpha-n \choose n}x^{\alpha-n}} $ implies
$ f^{(n)}(x)=n!\sum\limits_{k=0}^{\infty}{n-1-k \choose k}x^{-(k+1)}=
n!\sum\limits_{k=0}^{\infty}{-1-k-n \choose n}\frac{x^{-1-k-n}}{(k+n)!} .$ Because for $ z \in {\mathbb C} \; ,\; m \in {\mathbb N} $ we have $ {z \choose m}:= \frac{z(z-1)\cdots (z-m+1)}{m!}, $ that is ${-z \choose m}= (-1)^m{z-m-1 \choose m} $ , we may write
$ f^{(n)}(x)=  \dfrac{n!(-1)^n}{x^{n+1}}\sum\limits_{k=0}^{\infty}\displaystyle {n+k\choose k}\dfrac{x^{-k}}{(n+k)!}= \dfrac{(-1)^n}{x^{n+1}}\sum\limits_{k=0}^{\infty}\dfrac{x^{-k}}{k!}=
 \dfrac{(-1)^n}{x^{n+1}}\displaystyle e^{\frac{1}{x}}\; .                 $

 Профиль  
                  
Показать сообщения за:  Поле сортировки  
Начать новую тему Ответить на тему  [ Сообщений: 8 ] 

Модераторы: Модераторы Математики, Супермодераторы



Кто сейчас на конференции

Сейчас этот форум просматривают: нет зарегистрированных пользователей


Вы не можете начинать темы
Вы не можете отвечать на сообщения
Вы не можете редактировать свои сообщения
Вы не можете удалять свои сообщения
Вы не можете добавлять вложения

Найти:
Powered by phpBB © 2000, 2002, 2005, 2007 phpBB Group